Mathcenter Forum  

Go Back   Mathcenter Forum > คณิตศาสตร์โอลิมปิก และอุดมศึกษา > พีชคณิต
สมัครสมาชิก คู่มือการใช้ รายชื่อสมาชิก ปฏิทิน ข้อความวันนี้

ตั้งหัวข้อใหม่ Reply
 
เครื่องมือของหัวข้อ ค้นหาในหัวข้อนี้
  #1  
Old 03 กันยายน 2008, 19:06
Spotanus's Avatar
Spotanus Spotanus ไม่อยู่ในระบบ
จอมยุทธ์หน้าหยก
 
วันที่สมัครสมาชิก: 28 มีนาคม 2007
ข้อความ: 171
Spotanus is on a distinguished road
Default Functional Equation

Given a function $g:\mathbb{R}\rightarrow \mathbb{R}^{+}$. Show that there exist
(a) infinitely many functions $f:\mathbb{R}\rightarrow \mathbb{R}$ that
$$ \forall x\in \mathbb{R} ,\left(f\left(x\right)\right)^{2}=\left(g\left(x\right)\right)^{2}$$.
(b) exactly two functions $f:\mathbb{R}\rightarrow \mathbb{R}$ that
$$ \forall x,y\in \mathbb{R} ,f\left(x\right)f\left(y\right)=g\left(x\right)g\left(y\right)$$.
__________________
ในโลกนี้มีอสมการมากมายที่กระจายไม่ออก
ดังนั้นถ้ารู้ว่าตนกระจอกก็อย่าอาย
ถ้าอยากออกก็ต้องกระจาย จะได้ไม่ต้องอายที่ตนกระจอก


(Vasc's)
$$\left( a^{2}+b^{2}+c^{2} \right)^{2} \geq 3\left(a^{3}b+b^{3}c+c^{3}a\right)$$
ตอบพร้อมอ้างอิงข้อความนี้
  #2  
Old 03 ตุลาคม 2008, 21:58
Spotanus's Avatar
Spotanus Spotanus ไม่อยู่ในระบบ
จอมยุทธ์หน้าหยก
 
วันที่สมัครสมาชิก: 28 มีนาคม 2007
ข้อความ: 171
Spotanus is on a distinguished road
Default

ควรค่าแก่กาลเวลา และกาลเทศะ...
(a) เห็นได้ชัด
(b) อันดับแรกเราแทน $y=x$ เป็นจำนวนจริงใดใดซะก่อน ทีนี้เราจะได้ว่า
ทุก $x \in \mathbb{R}$, $\left|f\left(x\right)\right| =\left|g\left(x\right)\right|$
เนื่องจาก $g\left(x\right) \not= 0$ สำหรับทุก $x \in \mathbb{R}$
ดังนั้น $f\left(x\right) \not= 0$ สำหรับทุก $x \in \mathbb{R}$ ด้วย
ต่อไปพิจาณา $f\left(0\right)$
กรณีที่๑ ถ้า $sgn\left(f\left(0\right)\right)=sgn\left(g\left(0\right)\right)$
แทนลงในสมการหลักด้วย $\left(x,y\right)$ เป็น $\left(x,0\right)$ สำหรับ $x \in \mathbb{R}$ ใดใด
ดังนั้น $f\left(x\right) \equiv g\left(x\right)$
กรณีที่๒ ถ้า $sgn\left(f\left(0\right)\right)=-sgn\left(g\left(0\right)\right)$
จะได้ว่า $f\left(x\right) \equiv -g\left(x\right)$

ในนัยกลับกัน นำสองฟังก์ชันนี้กลับไปแทน จะได้ว่าฟังก์ชันทั้งสองสอดคล้อง
ดังนั้น มีสองฟังก์ชันพอดีที่สอดคล้อง
__________________
ในโลกนี้มีอสมการมากมายที่กระจายไม่ออก
ดังนั้นถ้ารู้ว่าตนกระจอกก็อย่าอาย
ถ้าอยากออกก็ต้องกระจาย จะได้ไม่ต้องอายที่ตนกระจอก


(Vasc's)
$$\left( a^{2}+b^{2}+c^{2} \right)^{2} \geq 3\left(a^{3}b+b^{3}c+c^{3}a\right)$$
ตอบพร้อมอ้างอิงข้อความนี้
ตั้งหัวข้อใหม่ Reply


หัวข้อคล้ายคลึงกัน
หัวข้อ ผู้ตั้งหัวข้อ ห้อง คำตอบ ข้อความล่าสุด
hard functional equation dektep พีชคณิต 6 14 เมษายน 2016 17:48
Equation Like Pell's Equation Anonymous314 ทฤษฎีจำนวน 11 07 มกราคม 2009 00:26
Functional Equation Problem RoSe-JoKer พีชคณิต 18 17 พฤษภาคม 2008 12:39
IMO;Functional Equation The jumpers พีชคณิต 4 12 พฤษภาคม 2008 14:43
Functional Equation dektep พีชคณิต 14 14 มีนาคม 2008 11:35


กฎการส่งข้อความ
คุณ ไม่สามารถ ตั้งหัวข้อใหม่ได้
คุณ ไม่สามารถ ตอบหัวข้อได้
คุณ ไม่สามารถ แนบไฟล์และเอกสารได้
คุณ ไม่สามารถ แก้ไขข้อความของคุณเองได้

vB code is On
Smilies are On
[IMG] code is On
HTML code is Off
ทางลัดสู่ห้อง


เวลาที่แสดงทั้งหมด เป็นเวลาที่ประเทศไทย (GMT +7) ขณะนี้เป็นเวลา 02:37


Powered by vBulletin® Copyright ©2000 - 2024, Jelsoft Enterprises Ltd.
Modified by Jetsada Karnpracha